- PowerScore Staff
- Posts: 5972
- Joined: Mar 25, 2011
- Sat Jan 21, 2012 12:00 am
#44051
Complete Question Explanation
(The complete setup for this game can be found here: lsat/viewtopic.php?t=16041)
The correct answer choice is (C)
This is a tricky question as well, and the question stem asks for you to identify the pair of variables where at least one of the two is always on the committee. Thus, if a scenario can be found where neither variable is on the committee, that answer choice is incorrect. In questions such as this, prior work is invaluable, so make sure to check the work you produced in solving the other questions in the game.
The hypothetical produced by the correct answer to question #12—J, K, M, Q, S—eliminates answer choice (A) because neither F nor P appears in the hypothetical. Other work in the game can be used to attack some of the other answers (#15 for answer choice (B), for example), but because many of those questions produced only partial hypotheticals, this can be challenging.
Let’s take a different attack, and look at each answer choice from a more abstract standpoint, one that assesses the conditional value of each variable:
Answer choice (A): contains two sufficient conditions
Answer choice (B): contains two sufficient conditions (one is also a necessary condition)
Answer choice (C): contains two necessary conditions
Answer choice (D): contains two sufficient conditions (one is also a necessary condition)
Answer choice (E): contains two randoms
In a game such as this one, where there is an Overloaded set of variables for the group being formed, variables that are necessary conditions are extremely powerful because when they are removed, another variable is typically also removed. Thus, answer choice (C) looks the most promising, followed by (B) and (D), then answer choice (A). answer choice (E) is the least likely to be correct as randoms are not likely to have to appear in every viable committee.
Given that (C) is the most promising answer, let’s start our analysis there. Answer choice (C) contains two representatives who are necessary for other representatives to be selected. So, eliminating both K and Q would also eliminate F and G from the committee. That leaves only J, M, P, R, and S for the committee. However, from the sixth rule, M and P cannot be on the same committee, so removing K and Q does not leave enough representatives to form a viable committee. Thus, either K or Q must be on every viable committee, and answer choice (C) is correct.
(The complete setup for this game can be found here: lsat/viewtopic.php?t=16041)
The correct answer choice is (C)
This is a tricky question as well, and the question stem asks for you to identify the pair of variables where at least one of the two is always on the committee. Thus, if a scenario can be found where neither variable is on the committee, that answer choice is incorrect. In questions such as this, prior work is invaluable, so make sure to check the work you produced in solving the other questions in the game.
The hypothetical produced by the correct answer to question #12—J, K, M, Q, S—eliminates answer choice (A) because neither F nor P appears in the hypothetical. Other work in the game can be used to attack some of the other answers (#15 for answer choice (B), for example), but because many of those questions produced only partial hypotheticals, this can be challenging.
Let’s take a different attack, and look at each answer choice from a more abstract standpoint, one that assesses the conditional value of each variable:
Answer choice (A): contains two sufficient conditions
Answer choice (B): contains two sufficient conditions (one is also a necessary condition)
Answer choice (C): contains two necessary conditions
Answer choice (D): contains two sufficient conditions (one is also a necessary condition)
Answer choice (E): contains two randoms
In a game such as this one, where there is an Overloaded set of variables for the group being formed, variables that are necessary conditions are extremely powerful because when they are removed, another variable is typically also removed. Thus, answer choice (C) looks the most promising, followed by (B) and (D), then answer choice (A). answer choice (E) is the least likely to be correct as randoms are not likely to have to appear in every viable committee.
Given that (C) is the most promising answer, let’s start our analysis there. Answer choice (C) contains two representatives who are necessary for other representatives to be selected. So, eliminating both K and Q would also eliminate F and G from the committee. That leaves only J, M, P, R, and S for the committee. However, from the sixth rule, M and P cannot be on the same committee, so removing K and Q does not leave enough representatives to form a viable committee. Thus, either K or Q must be on every viable committee, and answer choice (C) is correct.
Dave Killoran
PowerScore Test Preparation
Follow me on X/Twitter at http://twitter.com/DaveKilloran
My LSAT Articles: http://blog.powerscore.com/lsat/author/dave-killoran
PowerScore Podcast: http://www.powerscore.com/lsat/podcast/
PowerScore Test Preparation
Follow me on X/Twitter at http://twitter.com/DaveKilloran
My LSAT Articles: http://blog.powerscore.com/lsat/author/dave-killoran
PowerScore Podcast: http://www.powerscore.com/lsat/podcast/